Question and Answers Forum

All Questions      Topic List

Algebra Questions

Previous in All Question      Next in All Question      

Previous in Algebra      Next in Algebra      

Question Number 73036 by mathmax by abdo last updated on 05/Nov/19

calculate 1) Σ_(k=1) ^n  k^2 (n+1−k)  2)Σ_(1≤i≤j≤n)  ij

$$\left.{calculate}\:\mathrm{1}\right)\:\sum_{{k}=\mathrm{1}} ^{{n}} \:{k}^{\mathrm{2}} \left({n}+\mathrm{1}−{k}\right) \\ $$$$\left.\mathrm{2}\right)\sum_{\mathrm{1}\leqslant{i}\leqslant{j}\leqslant{n}} \:{ij} \\ $$

Commented by mathmax by abdo last updated on 06/Nov/19

1) Σ_(k=1) ^n k^2 (n+1−k) =(n+1)Σ_(k=1) ^n k^2  −Σ_(k=1) ^n  k^3   =(n+1)×((n(n+1)(2n+1))/6)−((n^2 (n+1)^2 )/4)  =((n(n+1))/2){(((n+1)(2n+1))/3)−((n(n+1))/2)}  =((n(n+1))/2){((2n^2 +3n+1)/3)−((n^2  +n)/2)}  =((n(n+1))/(12))(4n^2  +6n+2−3n^2 −3n)  =((n(n+1))/(12))(n^2 +3n+2) =((n(n+1)(n^2  +3n+2))/(12))

$$\left.\mathrm{1}\right)\:\sum_{{k}=\mathrm{1}} ^{{n}} {k}^{\mathrm{2}} \left({n}+\mathrm{1}−{k}\right)\:=\left({n}+\mathrm{1}\right)\sum_{{k}=\mathrm{1}} ^{{n}} {k}^{\mathrm{2}} \:−\sum_{{k}=\mathrm{1}} ^{{n}} \:{k}^{\mathrm{3}} \\ $$$$=\left({n}+\mathrm{1}\right)×\frac{{n}\left({n}+\mathrm{1}\right)\left(\mathrm{2}{n}+\mathrm{1}\right)}{\mathrm{6}}−\frac{{n}^{\mathrm{2}} \left({n}+\mathrm{1}\right)^{\mathrm{2}} }{\mathrm{4}} \\ $$$$=\frac{{n}\left({n}+\mathrm{1}\right)}{\mathrm{2}}\left\{\frac{\left({n}+\mathrm{1}\right)\left(\mathrm{2}{n}+\mathrm{1}\right)}{\mathrm{3}}−\frac{{n}\left({n}+\mathrm{1}\right)}{\mathrm{2}}\right\} \\ $$$$=\frac{{n}\left({n}+\mathrm{1}\right)}{\mathrm{2}}\left\{\frac{\mathrm{2}{n}^{\mathrm{2}} +\mathrm{3}{n}+\mathrm{1}}{\mathrm{3}}−\frac{{n}^{\mathrm{2}} \:+{n}}{\mathrm{2}}\right\} \\ $$$$=\frac{{n}\left({n}+\mathrm{1}\right)}{\mathrm{12}}\left(\mathrm{4}{n}^{\mathrm{2}} \:+\mathrm{6}{n}+\mathrm{2}−\mathrm{3}{n}^{\mathrm{2}} −\mathrm{3}{n}\right) \\ $$$$=\frac{{n}\left({n}+\mathrm{1}\right)}{\mathrm{12}}\left({n}^{\mathrm{2}} +\mathrm{3}{n}+\mathrm{2}\right)\:=\frac{{n}\left({n}+\mathrm{1}\right)\left({n}^{\mathrm{2}} \:+\mathrm{3}{n}+\mathrm{2}\right)}{\mathrm{12}} \\ $$

Commented by mr W last updated on 06/Nov/19

n^2 +3n+2=(n+1)(n+2)  ⇒we have the same result.

$${n}^{\mathrm{2}} +\mathrm{3}{n}+\mathrm{2}=\left({n}+\mathrm{1}\right)\left({n}+\mathrm{2}\right) \\ $$$$\Rightarrow{we}\:{have}\:{the}\:{same}\:{result}. \\ $$

Commented by mathmax by abdo last updated on 06/Nov/19

yes sir.

$${yes}\:{sir}. \\ $$

Answered by mr W last updated on 06/Nov/19

Σk^2 (n+1−k)  =(n+1)((n(n+1)(2n+1))/6)−((n^2 (n+1)^2 )/4)  =((n(n+1)^2 (n+2))/(12))    ΣΣij=Σi((n(n+1))/2)=((n^2 (n+1)^2 )/4)

$$\Sigma{k}^{\mathrm{2}} \left({n}+\mathrm{1}−{k}\right) \\ $$$$=\left({n}+\mathrm{1}\right)\frac{{n}\left({n}+\mathrm{1}\right)\left(\mathrm{2}{n}+\mathrm{1}\right)}{\mathrm{6}}−\frac{{n}^{\mathrm{2}} \left({n}+\mathrm{1}\right)^{\mathrm{2}} }{\mathrm{4}} \\ $$$$=\frac{{n}\left({n}+\mathrm{1}\right)^{\mathrm{2}} \left({n}+\mathrm{2}\right)}{\mathrm{12}} \\ $$$$ \\ $$$$\Sigma\Sigma{ij}=\Sigma{i}\frac{{n}\left({n}+\mathrm{1}\right)}{\mathrm{2}}=\frac{{n}^{\mathrm{2}} \left({n}+\mathrm{1}\right)^{\mathrm{2}} }{\mathrm{4}} \\ $$

Terms of Service

Privacy Policy

Contact: info@tinkutara.com